Herself32's Blog

「数学」- 暑假后文化课能力复健 20210830 ~ 20210905

序言

暑假荒废过去了,啥也没学到,也许学到了啥也没学明白,生活逐渐废柴化。

但由于目前无法公开的原因来到了「内卷之都」,所以感气氛而发,决定整一个数学文化课能力复健。

大概每天会从夏老师的模拟卷里挑一套,复健内容主要包括数列、圆锥曲线、函数导数的客观题、主观题,当然遇到不错的立体几何/概率统计也会随缘做一做,因为并不是给别人看的所以不会有图解,有的时候甚至不会有严谨过程,所以看的人麻烦把它当做一个无聊的人弄得 Problem List 就好,希望不会鸽。

upd on 20210905: 由于文本量有点恐怖,我自己看加载公式时间都已经达到 1min+ 了,所以决定一周发一个 post(

2021/08/30: 江西智学联盟体2022届高三年级第一次联考

圆锥曲线

\(T_{16}.\) 已知双曲线 \(C: \frac{y^2}{a^2} - \frac{x^2}{b^2} = 1(a > 0, b > 0)\) 的上、下焦点分别为 \(F_1,F_2\),过 \(F_1\) 的直线交双曲线上支于 \(A,B\) 两点,且满足 \(\overrightarrow{BF_1} = 4\overrightarrow{F_1A}, |AF_2| = |AB|\),则双曲线的离心率为 \(\underline{\quad\quad}\)


这题发现就是一堆边长,那么优先考虑双曲线第一定义,然后两遍余弦定理完事,复健第一天我也不会整出什么花活来难为自己不是。

考虑设 \(|F_1A| = x\),那么 \(|F_1B| = 4x\),那么 \(|AF_2| = |AB| = |AF_1| + |F_1B| = 5x\).

根据双曲线第一定义得到 \(|AF_2| - |AF_1| = 2a\),移个项可以得到 \(|AF_2| = 2a + |AF_1|\),把上面的结果代进去就有 \(4x = 2a \Rightarrow x = \frac 12 a\)

那么我们可以得到 \(\triangle ABF_2\) 全部的边长了: \(|AB| = |AF_2| = \frac 52a, |BF_2| = 4a\).

然后为了和 \(b,c\) 扯上关系,我们可以在 \(\triangle ABF_2\)\(\triangle AF_1F_2\) 中用两遍余弦定理去表示 \(\angle F_1AF_2\),这样就能得到离心率了.

\[\cos \angle AF_1F_2 = \frac{2(\frac 52a)^2 - (4a)^2}{2 \cdot (\frac 52a)^2} = \frac{(\frac 12a)^2 + (\frac 52a)^2 - (2c)^2}{2 \cdot (\frac 12a)(\frac 52a)} \]

解得 \(40 c^2 = 72 a^2 \Rightarrow e = \frac 35 \sqrt{5}\)


\(T_{20}.\) 过抛物线 \(y^2 = 2px(p > 0)\) 的焦点 \(F\) 做倾斜角为 \(\theta (\theta \neq \frac {\pi}{2})\) 的直线,交抛物线于 \(A,B\) 两点,当 \(\theta = \frac{\pi}{3}\) 时,以 \(FA\) 为直径的圆与 \(y\) 轴相切于点 \(T(0, \sqrt 3)\).

\((1).\) 求抛物线的方程.
\((2).\) 试问在 \(x\) 轴上是否存在一个定点 \(P\),使得 \(|FA| \cdot |PB| = |FB| \cdot |PA|\) 成立?若存在,求出 \(P\) 的坐标,若不存在,请说明理由.


这题听 xls 说实际就是俩结论,第一问白送,第二问换一换式子就能看出来角平分线第二定理,斜率和为 \(0\) 简单一算完事.

\((1).\)\(AF\) 中点 \(C\),连接 \(CT\),由题意 \(\odot C\)\(y\) 轴相切于 \(T\),所以 \(CT \perp y\) 轴.

因为 \(T(0, \sqrt 3)\),得 \(y_C = \sqrt 3\),又 \(\theta = \frac{\pi}{3}\),所以 \(|FC| = \frac{\sqrt 3}{\sin \theta} = 2 = |AC|\).

所以 \(|AF| = 4\),那么根据抛物线第一定义,\(A\) 到准线 \(x = -\frac p2\) 的距离 \(d = |AF| = 4 = \frac p2 + \frac p2 + |x_A - x_F| = p+2 = 4 \Rightarrow p = 2\).

所以抛物线方程为 \(y^2 = 4x\).

\((2).\) 假设存在 \(P(t,0)\) 满足 \(|FA| \cdot |PB| = |FB| \cdot |PA|\).
^4
\(AB: x = my+1, A(x_1, y_1), B(x_2, y_2)\),联立抛物线与直线方程可得:

\(y^2-4my-4=0 \Delta = 16m^2+16 > 0\)\(y_1 + y_2 = 4m, y_1y_2 = -4\).

因为 \(|FA| \cdot |PB| = |FB| \cdot |PA|\)\(\frac{|FA|}{|FB|} = \frac{|PA|}{|PB|}\),根据角平分线第二定理, \(PF\) 平分 \(\angle APB\).

所以 \(k_{AP} + k_{BP} = 0 \Rightarrow \frac{y_1}{x_1 - t} + \frac{y_2}{x_2 - t} = 0\).

化简得 \(4m(-t-1) = 0\),因为 \(\theta \neq \frac{\pi}{2}\),所以 \(m \neq 0\),所以 \(t = -1\),即存在 \(P(-1,0)\),满足 \(|FA| \cdot |PB| = |FB| \cdot |PA|\).

其实有这样一个更为广泛的结论,对于抛物线 \(y^2 = 2px(p > 0)\),若存在两个在 \(x\) 轴上且关于 \(y\) 轴对称的点 \(P_1,P_2\),有一条过 \(P_2\) 的直线与抛物线交于不同的两点 \(A,B\),那么 \(P_1P_2\) 平分 \(\angle AP_1B\).

函数导数

\(T_{12}.\) 已知命题 \(p: f(x) = \ln x-x^2+ax\) 在区间 \([1,+\infty)\) 上单调递减,命题 \(q: x \geq e\) 时,\(3x^4 \ln x \geq (a-x)e^{\frac ax-1}\) 恒成立,若 \(\lnot p \land q\) 为真命题,则实数 \(a\) 的取值范围是 \(\underline{\quad\quad}\).


这题就是个二合一,sub 1 没有难度,sub 2 是个指对混合的同构,说实话我觉得同构就是一类代数变形之后利用单调性,但当然还是需要点技巧的。

由题意知 \(\lnot p\)\(q\) 均为真命题. 注意 \(\lnot p\) 并不是 \(f(x)\) 在指定区间上单调递增,而是 单调递减,也就是导函数不恒负的意思.

那么 \(f'(x) = \frac 1x-2x+a = \frac{-2x^2+ax+1}{x}\),因为定义域 \(x>0\),那么符号取决于分子那个二次式,\(\Delta = a^2 + 8 > 0\),那么只需要 \(f'(1) > 0\) 就行了。

代入得 \(f'(1) = 1-2+a > 0 \Rightarrow a > 1\). 然后就是这个看起来非常炫酷的 sub 2.

发现这是一个指对混合的式子,直接处理实在是过于复杂,那么考虑同构化,首先把右面弄得整齐一点:

\[3x^3\ln x \geq \frac{a-x}{x}e^\frac{a-x}{x} \]

看到这里,我们充分有理由相信,最后同构出来的函数应当是 \(h(x) = xe^x\),那么按照这个方向把左边的式子做个对数运算,得到:

\[x^3\ln x^3 \geq \frac{a-x}{x}e^\frac{a-x}{x} \Leftrightarrow h(\ln x^3) \geq h \left(\frac{a-x}{x} \right) \]

显然 \(h(x)\) 是在定义域上单调递增的,那么原式等价于 \(3\ln x \geq \frac{a-x}{x} \Leftrightarrow a \leq 3x\ln x + x \Rightarrow a \leq 4e\). 综上 \(a \in (1, 4e]\).


\(T_{21}.\) 已知函数 \(f(x) = ae^{2x} - 2(ax+1)e^x, g(x) = x^2+2x\).

\((1).\)\(a < 0\) 时,讨论 \(f(x)\) 的单调性.
\((2).\)\(a \in (0,2]\) 时,证明: 函数 \(f(x)\)\(g(x)\) 的图像上恰有两对关于 \(x\) 轴对称的点(参考数据 \(\ln 2 \approx 0.6931\)


这题可以说是计算量巨大,其实第二问的最后一部分取点过程还是比较精彩的,但目前我实在是有心无力,就直接求个极限完事吧(

\((1).\) \(f'(x) = 2ae^{2x} - 2(ax+a+1)e^x = 2e^x(ae^x-ax-a-1)\). 因为 \(2e^x\) 恒正,考虑对后面的有效部分构造函数分析,令 \(f_1(x) = ae^x-ax-a-1\).

\(f_1'(x) = a(e^x - 1)\),因为 \(a < 0\),所以当 \(e^x - 1 = 0\)\(x = 0\)\(f_1(x)\) 取得极值.

\(x \in (-\infty,0)\) 时,\(f_1'(x) > 0\)\(f_1(x)\) 单调递增;当 \(x \in (0,+\infty)\) 时,\(f_1'(x) < 0\)\(f_1(x)\) 单调递减.

所以 \(f_1(x) \leq f_1(0) = -1 < 0\),所以 \(f'(x)\)\(\mathbb{R}\) 上恒负,\(f(x)\)\(\mathbb{R}\) 上单调递减.

\((2).\) 由题意,有且仅有两个不等的实数 \(x_1,x_2\),满足 \(f(x_1) = -g(x_1), f(x_2) = -g(x_2)\).

那么要证原命题,即证 \(h(x) = f(x) + g(x) = ae^{2x} - 2(ax+1)e^x + x^2 + 2x\)\(\mathbb{R}\) 上有且仅有两个零点.

\[h'(x) = 2ae^{2x} - 2(ax+a+1)e^x + 2x + 2 = 2(ae^x - 1)(e^x - x - 1) \]

易知 \(e^x-x-1\)\(\mathbb{R}\) 上恒非负,那么当 \(ae^x-1=0\)\(x = \ln \frac 1a\) 时,\(h'(x) = 0\)\(h(x)\) 取到极值.

\(x \in \left(-\infty,\ln \frac 1a \right)\) 时,\(h'(x) < 0\)\(h(x)\) 单调递减;当 \(x \in \left(\ln \frac 1a,+\infty \right)\) 时,\(h'(x) > 0\)\(h(x)\) 单调递增,所以

\[h(x) \geq h\left(\ln \frac 1a \right) = \frac 1a - \frac 2a\left(a\ln \frac 1a+1 \right) + \left( \ln \frac 1a\right)^2 + 2\ln \frac 1a \]

\(t = \frac 1a\),则 \(t \in [\frac 12, +\infty)\),构造函数 \(\varphi(t) = \ln^2 t-t\),则 \(\varphi'(t) = \frac{2\ln t-t}{t}\).

构造函数 \(\varphi_1(t) = 2\ln t - t\),则 \(\varphi_1'(t) = \frac{2-t}{t}\). 所以当 \(2-t=0\),即 \(t=2\) 时,\(\varphi_1(t)\) 取得极值.

\(t \in (\frac 12,2)\) 时,\(\varphi_1'(t) > 0\)\(\varphi_1(t)\) 单调递增;当 \(t \in (2,+\infty)\) 时,\(\varphi_1'(t) < 0\)\(\varphi_1(t)\) 单调递减,所以 \(\varphi_1(t) \leq \varphi_1(2) = 2(\ln 2 - 1) < 0\).

所以 \(\varphi'(t)\)\([\frac 12,+\infty)\) 上恒负,\(\varphi(t)\) 单调递减,所以 \(\varphi(t) \leq \varphi \left(\frac 12 \right) = \ln 2 - \frac 12 < (0.7)^2 - 0.5 < 0\),所以 \(h \left( \ln \frac 1a\right) < 0\).

\(x \to +\infty\) 时,\(h(x) \sim ae^{2x} - 2(ax+1)e^x = e^x(ae^x - 2ax - 2) > e^x[(e-a)x-2] \to \infty\),所以根据零点存在性定理,\(h(x)\)\(\left(\ln \frac 1a,+\infty \right)\) 上有一个零点.

\(x \to -\infty\) 时,取 \(x_0 = \min \{ -2, -\frac 1a \}\),则 \(h(x_0) > 0\),所以根据零点存在性定理,\(h(x)\)\(\left(x_0,\ln \frac 1a\right)\) 上有一个零点.

综上,原命题 \(h(x)\)\(\mathbb{R}\) 上有且仅有两个零点得证,\(Q.E.D\)

2021/08/31: 贵阳市2022届高三年级摸底考试+2021全国甲卷理数

数列 & 圆锥曲线

贵阳 \(T_{16}.\) 函数 \(y = x^2\) 在点 \((n,n^2) \,(n \in \mathbb{N}^+)\) 处的切线记为 \(l_n\),直线 \(l_n,l_{n+1}\)\(x\) 轴围成的三角形面积记为 \(S_n\),则数列 \(\{ \frac {1}{S_n}\}\) 的前 \(n\) 项和为 \(\underline{\quad\quad}\).


这题其实挺常规的,思路非常顺畅且自然,不过可以记住那个抛物线上任意一点切线在 \(x\) 轴上的截距是该点横坐标的一半的结论.

\(y' = 2x\),所以 \(l_n\) 的斜率为 \(2n\),随之可以写出 \(l_n\) 的方程: \(l_n: y = 2nx - n^2\),与 \(x\) 轴交点为 \((\frac n2,0)\).

同理我们可以得到 \(l_{n+1}\) 的方程: \(l_{n+1}: y = 2(n+1)x - (n+1)^2\),以及它与 \(x\) 轴的交点 \((\frac {n+1}{2}, 0)\),那么第 \(n\) 个三角形的底就是 \(\frac {n+1}{2} - \frac n2 = \frac 12\).

联立 \(l_n\)\(l_{n+1}\) 得到 \(2nx - n^2 = 2(n+1)x - (n+1)^2\),整理得 \(x = n + \frac 12\),代入直线方程得到 \(y = n(n+1)\),求面积得 \(S_n = \frac{4}{n(n+1)}\),那么.

\[\sum \frac{1}{S_n} = 4 \sum_{i=1}^n \frac{1}{i(i+1)} = 4 \left( 1 - \frac 12 + \frac 12 - \frac 13 + \dots + \frac 1n - \frac{1}{n+1}\right) = \frac{4n}{n+1} \]


甲卷 \(T_{18}.\) 已知数列 \(\{ a_n\}\) 的各项均为正数 \(a_2 = 3a_1\),记 \(S_n\) 为数列 \(\{ a_n\}\) 的前 \(n\) 项和,若数列 \(\sqrt{S_n}\) 为等差数列,求证: 数列 \(\{ a_n\}\) 是等差数列.


其实这题原本是个结构不良,我懒得去做三个组合,就随便挑了一个做,总体感觉也是过于常规,当然第二个大题也不可能太难不是(

注意到 \(\sqrt {S_2} = \sqrt{S_1} + d \Leftrightarrow \sqrt{a_1 + a_2} = \sqrt{a_1} + d \Leftrightarrow d = \sqrt{a_1}\).

那么 \(\sqrt{S_n} = \sqrt{S_1} + (n - 1)\sqrt{a_1} = n\sqrt{a_1} \Rightarrow S_n = n^2a_1\).

所以当 \(n \geq 2\) 时,\(a_n = S_n - S_{n-1} = a_1[n^2-(n-1)^2] = (2n-1)a_1\),当 \(n = 1\) 时符合上式.

所以 \(a_{n+1} - a_n = 2a_1\) 为定值,所以数列 \(\{ a_n\}\) 是等差数列.


贵阳 \(T_{21}.\) 已知 \(A(-4,0), B(4,0)\) 均为曲线 \(C\) 上的点,\(P\) 为曲线 \(C\) 上的任意一点,且满足直线 \(PA\)\(PB\) 的斜率之积为 \(-\frac 14\).

\((1).\) 求曲线 \(C\) 的方程.
\((2).\) 斜率不为 \(0\) 的直线 \(l\) 过点 \(F(2\sqrt 3,0)\) 且与曲线 \(C\) 交于 \(M,N\) 两点,点 \(Q(0, \frac 32\sqrt{3})\),若 \(|MQ| = |NQ|\),求直线 \(l\) 的方程.


sub 1就是椭圆第三定义,sub 2利用等腰三角形三线合一算个垂直斜率积为 \(-1\) 结束,感觉这套题适合睡觉时候做.

\((1).\)\(P(x,y)\),由题意可得 \(\frac{y}{x-4} \cdot \frac{y}{x+4} = -\frac 14\),整理得 \(C:\frac{x^2}{16}+\frac{y^2}{4}=1\).

\((2).\)\(MN\) 中点 \(D\),因为 \(|MQ| = |NQ|\),所以 \(QD \perp MN\), 所以 \(k_{QD} \cdot k_{MN} = -1\)

\(MN: x=ty+2\sqrt 3, M(x_1, y_1), N(x_2, y_2)\),所以 \(D \left( \frac{x_1+x_2}{2}, \frac{y_1+y_2}{2}\right)\).

联立 \(MN\) 和曲线 \(C\) 可得 \((t^2+4)y^2+4\sqrt 3ty-4=0, \Delta = 64(t^2+1) > 0\),所以 \(y_1+y_2 = \frac{-4\sqrt 3}{t^2+4}, y_1y_2=\frac{-4}{t^2+4}\),所以.

\[k_{QD} = \frac{\frac 32\sqrt3 - y_1 - y_2}{-x_1-x_2} = \frac{\frac 32\sqrt 3 + \frac{2\sqrt 3t}{t^2+4}}{\frac{-8\sqrt3}{t^2+4}} = -t \]

化简得 \(\frac 32(t^2+4)+2t=8t \Rightarrow t = 2\),所以 \(l: x-2y-2\sqrt 3 = 0\).


甲卷 \(T_{20}.\) 抛物线 \(C\) 的顶点与坐标原点 \(O\) 重合,焦点在 \(x\) 轴上,直线 \(l:x=1\) 与抛物线交于 \(P,Q\) 两点,且 \(OP \perp OQ\). 已知点 \(M(2,0), \odot M\)\(l\) 相切.

\((1).\)\(C\)\(\odot M\) 的方程.
\((2).\)\(A_1,A_2,A_3\)\(C\) 上的三个点,直线 \(A_1A_2,A_1A_3\) 均与 \(\odot M\) 相切,判断直线 \(A_2A_3\)\(\odot M\) 的位置关系并说明理由.


其实这个题和八省联考的一道选择题类似,但由于八省联考的时候我没有太多关注,所以套路还是从这个题学会的.

\((1).\) 设抛物线 \(C\) 的方程为 \(y^2 = 2px(p > 0)\),由题意知,抛物线过点 \((1,1)\),代入得 \(1 = 2p \Rightarrow p = \frac 12\),所以抛物线方程为 \(y^2 = x\).

因为 \(\odot M\)\(x=1\) 相切,所以 \(\odot M\) 半径为 \(r = 1\),那么 \(\odot M\) 的方程为 \((x-2)^2+y^2 = 1\).

\((2).\)\(A_1A_2,A_1A_3\) 中有一条直线和 \(x\) 轴垂直时,不妨假设这条直线是 \(A_1A_2\),显然 \(A_1A_2\)\(x = 1\) 不重合,因为点 \((1,1)\) 引出 $\odot M $ 的另一条切线 \(y=1\) 和抛物线无交点.

所以 \(A_1A_2: x = 3 \Rightarrow A_1(3, \sqrt 3),A_2(3,-\sqrt 3)\).由题意知,\(A_2A_3: x+\sqrt 3y=0\),圆心 \(M\)\(A_2A_3\) 的距离为 \(\frac{|2|}{\sqrt{1+3}} = 1\),所以 \(\odot M\)\(A_2A_3\) 相切.

\(A_1A_2,A_1A_3\) 均与 \(x\) 轴不垂直时,设 \(A_1(y_1^2,y_1),A_2(y_2^2,y_2),A_3(y_3^2,y_3)\),所以 \(k_{A_1A_2} = \frac{y_2 - y_1}{y_2^2-y_1^2}\).

那么 \(A_1A_2: y = \frac{1}{y_1+y_2}(x-y_1^2)+y_1\),整理得 \(A_1A_2: x-(y_1+y_2)y+y_1y_2 = 0\),同理得 \(A_1A_3: x-(y_1+y_3)y+y_1y_3=0\).

因为 \(A_1A_2,A_1A_3\) 均与 \(\odot M\) 相切,所以

\[d_{M \to A_1A_2} = \frac{|2+y_1y_2|}{\sqrt{1+(y_1+y_2)^2}} = 1 \Leftrightarrow (2+y_1y_2)^2=1+(y_1+y_2)^2 \]

同理可得 \((2+y_1y_3)^2=1+(y_1+y_3)^2\),把这两条式子展开,我们得到

\[\begin{cases} (y_1^2-1)y_2^2+2y_1y_2+3-y_1^2=0 \\ \\ (y_1^2-1)y_3^2+2y_1y_3+3-y_1^2=0 \end{cases} \]

\(\color{red}{那么\; y_2,y_3 \;是方程\; (y_1^2-1)x^2+2y_1x+3-y_1^2=0 \;的两个解}\),所以 \(y_2+y_3=\frac{-2y_1}{y_1^2-1}, y_2y_3 = \frac{3-y_1^2}{y_1^2-1}\).

由上面 \(A_1A_2\) 的方程同理可得 \(A_2A_3: x-(y_2+y_3)y+y_2y_3=0\),所以圆心 \(M\) 到其的距离为

\[d_{M \to A_2A_3} = \frac{|2+y_2y_3|}{\sqrt{1+(y_2+y_3)^2}} = \frac{|\frac{y_1^2+1}{y_1^2-1}|}{\sqrt{\frac{4y_1^2}{(y_1^2-1)^2}+1}} = 1 \]

所以 \(\odot M\)\(A_2A_3\) 相切,综上\(\odot M\)\(A_2A_3\) 相切.

函数导数

贵阳 \(T_{20}.\) [加强版] 已知函数 \(f(x) = (a+1)\ln x+x-\frac ax, a \in R\).

\((1).\) 讨论函数 \(f(x)\) 的单调区间并求其最值.
\((2).\)\(a < 0\) 时,记 \(f(x)\) 的最小值为 \(g(a)\),求满足 \(g(a) > 2\)\(a\) 的范围.


这题出的实在是太拉了……,xls 讲第二问还没第一问时间长,所以我就斗胆推测了一下出题人原意,放个加强版上来(不过好像也没有加强到哪里去).

\((1).\) \(f(x) = (a+1)\ln x+x-\frac ax\),则 \(f'(x) = \frac{a+1}{x}+1+\frac{a}{x^2} = \frac{x^2+(a+1)x+a}{x^2} = \frac{(x+1)(x+a)}{x^2}\) .

所以,当 \(a \geq 0\) 时,\(f'(x)\)\((0, +\infty)\) 上恒正,\(f(x)\)\((0, +\infty)\) 上单调递增,没有最值.

\(a < 0\) 时,当 \(x \in (0,-a)\) 时,\(f'(x) < 0\)\(f(x)\) 单调递减,当 \(x \in (-a,+\infty)\) 时,\(f'(x)>0\)\(f(x)\) 单调递增.

此时 \(f(x)\)\(x=-a\) 处取得最小值 \(f(-a) = (a+1)\ln (-a)-a+1\).

\((2).\)\((1)\)\(g(a) = (a+1)\ln (-a)-a+1\),原题即求 \(g(a)>2 \Leftrightarrow (a+1)\ln (-a)-a-1>0\) 的解集.

原不等式等价于 \((a+1)[\ln(-a)-1] > 0\),所以可以得到两个括号内的部分都为正或都为负,解得 \(a \in (-e, -1)\).


甲卷 \(T_{21}\) 已知 \(a > 0\)\(a \neq 1\),函数 \(f(x) = \frac{x^a}{a^x}(x>0)\).

\((1).\)\(a=2\) 时,求 \(f(x)\) 的单调区间.
\((2).\) 若曲线 \(y = f(x)\) 与直线 \(y=1\) 有且仅有两个交点,求 \(a\) 的取值范围.


总之感觉还是比较常规,掺杂一点小技巧,毕竟甲卷的题还是比较适合吃饭时候做的(当然我现在这个水平还是要打起精神来的

\((1).\) \(f(x) = \frac{x^2}{2^x}\),所以 \(f'(x) = \frac{2x(2^x)-x^2(2^x\ln 2)}{(2^x)^2} = \frac{x(2 - \ln 2x)}{2^x}\).

所以当 \(2 - \ln 2x = 0\),即 \(x = \frac{2}{\ln 2}\) 时,\(f'(x) = 0\)\(f(x)\) 取得极值.

\(x \in (0, \frac{2}{\ln 2})\) 时,\(f'(x) > 0\)\(f(x)\) 单调递增,当 \(x \in (\frac{2}{\ln 2}, +\infty)\) 时,\(f'(x) < 0\)\(f(x)\) 单调递减.

\((2).\) 由题意知,方程 \(\frac{x^a}{a^x} = 1 \Leftrightarrow x^a = a^x \Leftrightarrow \color{red}{a\ln x = x \ln a} \Leftrightarrow \color{red}{\frac{\ln x}{x} = \frac{\ln a}{a}}\) 有且仅有两个实数根.

构造函数 \(g(x) = \frac{\ln x}{x}\),则 \(g'(x) = \frac{1 - \ln x}{x^2}\),所以当 \(1-\ln x=0\)\(x = e\) 时,\(g'(x)=0\)\(g(x)\) 取得极值.

\(x \in (0, e)\) 时,\(g'(x) > 0\)\(g(x)\) 单调递增,当 \(x \in (e, +\infty)\) 时,\(g'(x) < 0\)\(g(x)\) 单调递减.

又因为 \(\lim \limits_{x \to -\infty} g(x) = -\infty\)\(g(1) = 0, g(e) = \frac 1e\)\(\lim \limits_{x \to +\infty} g(x) = 0\),根据 \(g(x)\) 图像可得,当 \(\frac{\ln a}{a} \in (0, \frac 1e)\) 时,原方程有且仅有两个不等的实数根,解得 \(a \in (1, e) \cup (e, +\infty)\).

2021/09/01: 2021 全国新高考 II 卷

圆锥曲线

\(T_{21}.\) 已知椭圆 \(C: \frac{x^2}{a^2} + \frac{y^2}{b^2} = 1(a>0, b>0)\),右焦点 \(F(\sqrt 2,0)\),且离心率为 \(\frac {\sqrt 6}{3}\).

\((1).\) 求椭圆 \(C\) 的方程.
\((2).\)\(M,N\) 是椭圆 \(C\) 上的两点,且 \(MN\) 与曲线 \(\Sigma:x^2+y^2=b^2(x>0)\) 相切,证明: \(M,N,F\) 三点共线的充要条件是 \(|MN| = \sqrt 3\).


今天上午放过去了,所以今天提早下班(本来还有个二进制想说几句,但是对于 MOer/OIer 来说过于简单就⑧说了,圆锥曲线也是意料之中的简单(

\((1).\) 由题意知 \(c = \sqrt 2, \frac ca = \frac{\sqrt 6}{3}\),所以 \(a = \sqrt 3, b = \sqrt{a^2-c^2} = 1\),所以 \(C: \frac{x^2}{3} + y^2=1\).

\((2).\) 考虑证明必要性,设 \(MN: x=ty+\sqrt 2, M(x_1,y_1),N(x_2,y_2)\),当 \(t\) 不存在时,显然 \(MN\) 不与 \(\Sigma\) 相切.

那么联立 \(MN\)\(C\) 的方程,得到: \((ty+\sqrt 2)^2+3y^2-3=0 \Leftrightarrow (t^2+3)y^2-2\sqrt 2ty-1=0, \Delta = 12(t^2+1)>0\).

因为 \(MN\) 与曲线 \(\Sigma\) 相切,那么 \(d_{O \to MN} = \frac{|-\sqrt 2|}{\sqrt{t^2+1}} = 1 \Rightarrow t= \pm 1\).

代入联立得到的方程得,\(4y^2 \pm 2\sqrt 2y-1=0, \Delta = 24\),所以 \(|MN| = \sqrt{1+t^2}|y_1-y_2| = \sqrt{1+t^2}\frac{\sqrt{\Delta}}{|a|} = \sqrt 3\),成立.

再证充分性,设 \(MN: x=my+n, M(x_1,y_1),N(x_2,y_2)\),当 \(m\) 不存在时,显然 \(MN\) 不与 \(\Sigma\) 相切.

那么联立 \(MN\)\(C\) 的方程,得到 \((my+n)^2+3y^2-3=0 \Leftrightarrow (m^2+3)y^2+2mny-3+n^2=0, \Delta=4(2m^2n^2-3m^2+3n^2-9)\).

因为 \(MN\) 与曲线 \(\Sigma\) 相切,那么 \(d_{O \to MN} = \frac{|-n|}{\sqrt{1+m^2}} = 1 \Rightarrow n^2 = 1+m^2\),所以 \(\Delta = 8(m^4+m^2-3)\)

所以 \(|MN| = \sqrt{1+m^2} \frac{\sqrt{\Delta}}{|a|} = \sqrt 3 \Rightarrow m = \pm 1, n = \pm \sqrt 2\),又因为直线 \(x-y+\sqrt 2=0\) 和直线 \(x+y+\sqrt 2=0\) 均不与曲线 \(\Sigma\) 相切,所以舍去,那么 \(n=\sqrt 2\),所以 \(MN\) 过点 \(F(\sqrt 2,0)\),即 \(M,N,F\) 三点共线.

综上 \(M,N,F\) 三点共线的充要条件是 \(|MN| = \sqrt 3\).

函数导数

\(T_{16}.\) 已知函数 \(f(x) = |e^x-1|, x_1<0, x_2>0\),函数 \(f(x)\) 的图像在点 \(A(x_1,f(x_1)), B(x_2,f(x_2))\) 处的两条切线相互垂直,且分别交 \(y\) 轴于 \(M,N\) 两点,则 \(\frac{|AM|}{|BN|}\) 的取值范围是 \(\underline{\quad\quad}\).


感觉出题思路挺好的,要考察的点也没有像很多题一样明摆着,不过解题思路也是挺顺畅的(

\(f'(x) = {\rm sgn}(e^x-1)\cdot e^x\),所以 \(k_{AM} = f'(x_1) = -e^{x_1}, k_{BN} = f'(x_2) = e^{x_2}\),又因为 \(AM \perp BN\),所以 \(k_{AM} \cdot k_{BN} = -1 \Rightarrow e^{x_1+x_2}=1 \Rightarrow x_1+x_2=0\).

\[\frac{|AM|}{|BN|} = \color{red}{\frac{\sqrt{1+k_{AM}^2}|x_1-x_M|}{\sqrt{1+k_{BN}^2}|x_2-x_N|}} = \frac{\sqrt{1+e^{2x_1}}\cdot -x_1}{\sqrt{1+e^{2x_2}} \cdot x_2} = \sqrt {\frac {1+e^{-2x_2}}{1+e^{2x_2}}} = e^{-x_2} \in (0,1) \]


\(T_{22}.\) 已知函数 \(f(x) = (x-1)e^x-ax^2+b\).

\((1).\) 讨论 \(f(x)\) 的单调性.
\((2).\)\(a \in (0, \frac 12), b \leq 2a\),试证明: \(f(x)\) 有且仅有一个零点.


说实话这个题属于压轴题里面的简单题了,第二问的找点也没有前天做的那么难,作为高考卷还是很友好的(这 nm 就是你第一问要写这么多字的理由吗!

\((1).\) \(f'(x) = xe^x-2ax = x(e^x-2a)\),所以当 \(a \leq 0\) 时,当 \(x = 0\) 时,\(f'(x)=0\)\(f(x)\) 取得极值.

\(x \in (-\infty, 0)\) 时,\(f'(x) < 0\)\(f(x)\) 单调递减,当 \(x \in (0, +\infty)\) 时,\(f'(x) > 0\)\(f(x)\) 单调递增.

\(a > 0\) 时,若 \(\ln 2a = 0\)\(a = \frac 12\),此时 \(f'(x)\)\(\mathbb{R}\) 上恒正,所以 \(f(x)\)\(\mathbb{R}\) 上单调递增.

\(\ln 2a < 0\),即 \(0 < a < \frac 12\) 时,当 \(x = 0\)\(x = \ln 2a\) 时,\(f'(x)=0\)\(f(x)\) 取得极值.

\(x \in (-\infty, \ln 2a)\) 时,\(f'(x) > 0\)\(f(x)\) 单调递增,当 \(x \in (\ln 2a, 0)\) 时,\(f'(x) < 0\)\(f(x)\) 单调递减,当 \(x \in (0, +\infty)\) 时,\(f'(x) > 0\)\(f(x)\) 单调递增.

\(\ln 2a > 0\)\(a > \frac 12\) 时,当 \(x = 0\)\(x = \ln 2a\) 时,\(f'(x)=0\)\(f(x)\) 取得极值.

\(x \in (-\infty, 0)\) 时,\(f'(x) > 0\)\(f(x)\) 单调递增,当 \(x \in (0, \ln 2a)\) 时,\(f'(x) < 0\)\(f(x)\) 单调递减,当 \(x \in (\ln 2a, +\infty)\) 时,\(f'(x) > 0\)\(f(x)\) 单调递增.

\((2).\) \(f(0) = b-1 \leq 2a-1 < 0\),所以需证明 \(f(\ln 2a) = 2a(\ln 2a-1)-a(\ln 2a)^2+b = a\ln 2a(2 - \ln 2a)+b-2a \leq a\ln 2a(2 - \ln 2a) < 0\) 即可.

又因为 \(a > 0, \ln 2a \in (-\infty, 0), 2 - \ln 2a \in (2, +\infty)\),所以 \(f(\ln 2a) < 0\),所以只需证 \(\lim\limits_{x \to +\infty} f(x) = +\infty\) 即可.

\(x_0 = \max \{ 1, \sqrt{2-2b}\}\),则 \(f(x_0) = (x_0-1)e^{x_0}-ax_0^2+b > (x_0-1)(x_0+1)-\frac 12x_0^2+b = \frac 12 x_0^2-1+b \geq \frac 12(2-2b)-1+b = 0\),所以 \(f(x_0) \geq 0\).

根据零点存在性定理,\(f(x)\)\((0, x_0)\) 上有且仅有一个零点,又因为 \(f(0) < 0, f(\ln 2a)<0\),所以 \(f(x)\) 在其定义域上有且仅有一个零点.

2021/09/02: 2021 新高考 I 卷

数列

\(T_{16}.\) [加强版] 某校学生在研究民间剪纸艺术时,发现剪纸经常会沿纸的某条对称轴把纸对折,规格为 \(20 {\rm \;dm} \times 12 {\rm \;dm}\) 的长方形纸,对折 \(1\) 次一共可以得到 \(10 {\rm \;dm} \times 12 {\rm \;dm}\)\(20 {\rm \;dm} \times 6 {\rm \;dm}\) 两种规格的图形,它们的面积之和为 \(240 {\rm \;dm^2}\),对折 \(2\) 次一共可以得到 \(5 {\rm \;dm} \times 12 {\rm \;dm}\)\(10 {\rm \;dm} \times 6 {\rm \;dm}\) 以及 \(20 {\rm \;dm} \times 3 {\rm \;dm}\) 三种规格的图形,它们的面积之和为 \(180 {\rm \;dm^2}\). 那么记 \(a {\rm \;dm} \times b {\rm \; dm} \quad (a \neq b)\) 的折纸对折 \(k\) 次时所得所有规格图形的面积和为 \(S_k\),则 \(\sum \limits_{k=1}^n S_k = \underline{\quad\quad\quad}\).


原题直接对 \(20 \times 12\) 的长方形分析了,我感觉那样就变成一个 \(\rm trivial\) 的找规律了(而且目前网络上的解法大部分也是这样,所以就 ex 到 \(a \times b\) 了.

我们来对找规律的做法进行一下证明,定义 \(f(n)\) 表示该图形折叠 \(n\) 次时会得到的图形规格数,首先证明: 当 \(a \neq b\) 时,\(f(n) = n+1\).

我们来考虑第 \(n\) 次折叠的结果是什么,显然它们是 \(\left( \frac{a}{2^i}, \frac{b}{2^{n-i}}\right) \;(i = 0,1,2,\cdots,n)\),从 \(i\) 的计数就可以看出来了,显然 \(f(n) = n+1\).

显然,每进行一次折叠,图形面积会缩小一半,所以 \(S_k = f(k) \cdot \frac{ab}{2^k} = (k+1) \frac{ab}{2^k}\),显然这是一个差比混合的数列,求和就直接错位相减,得到

\[\sum_{k=1}^n S_k = \sum_{k=1}^n (k+1) \frac{ab}{2^k} = ab\left(3 + \frac{k-1}{2^k} \right) \]

圆锥曲线

\(T_{21}.\) 在平面直角坐标系 \(xOy\) 中,已知点 \(F_1(-\sqrt {17},0), F_2(\sqrt {17},0)\),点 \(M\) 满足 \(|MF_1| - |MF_2| = 2\),记 \(M\) 的轨迹为 \(C\).

\((1).\)\(C\) 的方程.
\((2).\) 设点 \(T\) 在直线 \(x = \frac 12\) 上,过 \(T\) 的两条直线分别交 \(C\)\(A,B\) 两点和 \(P,Q\) 两点,且 \(|TA| \cdot |TB| = |TP| \cdot |TQ|\),求直线 \(AB\) 的斜率与直线 \(PQ\) 的斜率之和.


我心心念念的四点共圆❤,其实这个题考场上一紧张忘了圆幂定理或者一激动忘了圆幂定理咋证也没有关系,弦长公式硬刚也是可以的捏~

\((1).\) 通过条件知道这是双曲线的一支,所以 \(c = \sqrt {17}, 2a = 2, b = \sqrt{c^2-a^2} = 4\),所以 \(C: x^2-\frac{y^2}{16}=1(x\geq 1)\).

\((2).\) 嗯,其实我在题目刚出来的时候就注意到这个可爱的四♀点♀共♀圆了捏❤,当时第一眼看到压轴是个考烂了的极值点偏移的破心情瞬间好起来了捏❤,但是缺点还是太结论了 TAT

再歪个楼说两句,在一年半之前 \(Dylan\) 大爷在 zhihu 上就发过圆锥曲线中四点共圆的详细解析,所以如果真的有人看这篇文章的话可以去学习一个 \(\to\) link.

\(T(\frac 12, m)\),因为 \(T\)\(AB\) 上,设直线 \(AB\) 的倾斜角为 \(\alpha\),则其参数方程为: \(\begin{cases} x = \frac 12+t\cos \alpha \\ y = m + t\sin \alpha\end{cases}\),同理,设直线 \(PQ\) 的倾斜角为 \(\beta\),则其参数方程为 \(\begin{cases} x = \frac 12+t\cos \beta \\ y = m + t\sin \beta\end{cases}\)

那么带入 \(C\) 的方程整理一下得到,\((16\cos^2 \alpha - \sin^2 \alpha)t^2+(16\cos \alpha-2m\sin \alpha)t-(m^2+12)=0\),所以 \(t_1t_2 = \frac{-m^2-12}{16\cos^2 \alpha - \sin^2 \alpha}\).

同理 \(t_3t_4 = \frac{-m^2-12}{16\cos^2 \beta-\sin^2 \beta}\),根据参数方程的几何意义,\(|TA|\cdot|TB|=|TP|\cdot|TQ| \Leftrightarrow t_1t_2=t_3t_4\),所以

\[\frac{-m^2-12}{16\cos^2 \alpha - \sin^2 \alpha} = \frac{-m^2-12}{16\cos^2 \beta-\sin^2 \beta} \Rightarrow 17\cos^2 \alpha - 1 = 17\cos^2 \beta - 1 \Rightarrow \alpha + \beta = \pi \]

所以,\(k_{AB} + k_{PQ} = \tan \alpha + \tan \beta = 0\).

[方法二 超纲 ] 考虑设 \(AB: k_1x-y+b_1=0, PQ: k_2x-y+b_2=0\),那么方程 \(\Sigma:(k_1x-y+b_1)(k_2x-y+b_2)=0\) 代表 \(AB\)\(PQ\) 上的所有点.

因为 \(|TA|\cdot|TB| = |TP| \cdot |TQ|\),所以 \(A,B,P,Q\) 四点共圆,那么考虑 \(\Sigma\)\(C\) 构成的交点的曲线系方程:

\[(k_1x-y+b_1)(k_2x-y+b_2) + \lambda \left( x^2 - \frac{y^2}{16} - 1\right) = 0 \]

观察得到 \(xy\) 项的系数为 \(-(k_1+k_2)\),因为 \(A,B,P,Q\) 四点共圆,所以 \(xy\) 项系数应为 \(0\),即 \(k_1+k_2=0\).

函数导数

\(T_{21}.\) 已知函数 \(f(x) = x(1-\ln x)\).

\((1).\) 讨论 \(f(x)\) 的单调性.
\((2).\)\(a,b\) 为两个不相等的正数,且 \(b\ln a - a\ln b = a-b\),求证: \(2 < \frac 1a+\frac 1b < e\).


真的没啥好说的 orz,左边就是个考的烂透了的极值点偏移,右边用极限的话也是极值点偏移,不用极限的话就是简单放缩(((

\((1).\) 由题意知 \(f'(x) = -\ln x\),所以当 \(x = 1\) 时,\(f'(x) = 0\)\(f(x)\) 取得极值.

\(x \in (0,1)\) 时,\(f'(x) > 0\)\(f(x)\) 单调递增,当 \(x \in (1, +\infty)\) 时,\(f'(x) < 0\)\(f(x)\) 单调递减.

\((2).\) 由题意知 \(b\ln a - a\ln b = a-b \Leftrightarrow \frac 1a \ln a - \frac 1b \ln b = \frac 1b - \frac 1a \Leftrightarrow \frac 1a(1 - \ln \frac 1a) = \frac 1b(1 - \ln \frac 1b)\).

所以令 \(\frac 1a = x_1, \frac 1b = x_2\),那么 \(f(x_1) = f(x_2)\).

\((1)\) 知,\(f(x)\)\((0,1)\) 上单调递增,在 \((1, +\infty)\) 上单调递减,所以 \(f(x) \leq f(1) = 1\).

\(a,b \in \mathbb{R}^+\)\(\forall x \in (0,1) f(x) > 0, f(e) = 0\),所以 \(x_1,x_2 \in (0, e)\),不妨设 \(x_1 \in (0, 1)\),则 \(x_2 \in (1, e)\).

先证 \(x_1 + x_2 > 2\),要证该不等式,即证 \(x_2 > 2 - x_1\),因为 \(x_2 > 1, 2-x_1 > 1\)\(f(x)\)\((1, +\infty)\) 上单调递减,所以即证 \(f(x_2) < f(2-x_1)\).

即证 \(f(x_1) < f(2-x_1)\),考虑令 \(g(x_1) = f(x_1) - f(2-x_1)\),所以 \(g'(x_1) = -\ln x_1-\ln (2-x_1) = -\ln [x_1(2-x_1)] > 0\),所以 \(g(x_1)\)\((0,1)\) 上单调递增,所以 \(g(x) < g(1) = 0\).

所以 \(f(x_1) < f(2-x_1)\),那么原不等式 \(x_1 + x_2 > 2\) 成立. 再证右边 \(x_1 + x_2 < e\).

因为 \(x_1 \in (0,1)\),所以 \(1 - \ln x_1 > 1\),所以 \(x_1(1-\ln x_1) > x_1\),那么 \(x_1 + x_2 < x_1(1-\ln x_1) + x_2 = x_2(1 - \ln x_2) + x_2\).

构造函数 \(h(x) = x(1 - \ln x)+x, x \in (1, e)\),所以 \(h'(x) = 1-\ln x>0\),所以 \(h(x)\)\((1,e)\) 上单调递增,那么 \(h(x) < h(e) = e\).

所以 \(x_1 + x_2 < e\),综上 \(2 < \frac 1a + \frac 1b < e\),证毕.

2021/09/03: 2021 全国乙卷

圆锥曲线

\(T_{21}.\) 已知抛物线 \(C: x^2 = 2py(p > 0)\) 的焦点为 \(F\),且 \(F\)\(\odot M: x^2 + (y+4)^2 = 1\) 上的点的距离的最小值为 \(4\).

\((1).\)\(p\).
\((2).\) 若点 \(P\)\(M\) 上,\(PA, PB\)\(C\) 的两条切线,\(A,B\) 是切点,求 \(\triangle PAB\) 面积的最大值.


阿基米德三角形嘛,总体来讲在近年来的模拟题里面出现的越来越频繁了,这题稍微推陈出新了一点(?,不过硬刚还是无问题的(

\((1).\) 由题意知 \(F(0, \frac p2)\),又因为 \(F\)\(\odot M: x^2 + (y+4)^2 = 1\) 上的点的距离的最小值为 \(4\),所以 \(F(1,0)\),所以 \(p = 2\).

\((2).\)\(A(x_1, \frac 14x_1^2), B(x_2, \frac 14x_2^2)\),由 \((1)\) 知,\(y = \frac 14x^2\),所以 \(y' = \frac 12x\).

所以 \(PA: y = \frac 12 x_1 - \frac 14x_1^2, PB: y = \frac 12 x_2 - \frac 14 x_2^2\),联立 \(PA\)\(PB\) 解析式可得 \(P \left(\frac {x_1+x_2}{2},\frac{x_1x_2}{4} \right)\).

\(AB: y = kx+b\),与抛物线方程联立可得,\(x^2 - 4kx - 4b=0\),所以 \(x_1 + x_2 = 4k, x_1x_2 = -4b\),所以 \(P \left(2k, -b\right)\).

\[|AB| = \sqrt{1+k^2} |x_A - x_B| = \sqrt{1+k^2} \frac{\sqrt{\Delta}}{|a|} = 4 \sqrt {1+k^2} \cdot \sqrt{k^2+b}, d_{P \to AB} = \frac{2|k^2+b|}{\sqrt{1+k^2}} \]

所以 \(S_{\triangle PAB} = \frac 12|AB|d_{P \to AB} = 4(k^2+b)^{\frac 32}\).

又因为 \(P\)\(\odot M\) 上,所以 \(4k^2 + (b-4)^2 = 1 \Rightarrow k^2 = \frac{-b^2 + 8b - 15}{4}\),代入得 \(S_{\triangle PAB} = 4 \left( \frac{-b^2+8b-15}{4}\right)^{\frac 32}\).

又因为 \(b \in (3,5)\),所以当 \(b = 5\) 时,\(S_{\triangle PAB}\) 取得最大值 \(S_{max} = 20 \sqrt 5\).

函数导数

\(T_{20}.\) [改编] 已知函数 \(f(x) = \ln (a-x)\),已知 \(x = 0\) 是函数 \(y = xf(x)\) 的极值点.

\((1).\) 求实数 \(a\) 的值.
\((2).\)\(g(x) = \frac{x+f(x)}{xf(x)}\),证明: \(0 < g(x) < 1\).


本来第二问是只有右边的,我一开始随便划拉的时候把左边证出来了,但难度又不增加多少,所以就叫做「改编」而非「加强」了吧(

\((1).\) 由题意知,\(y = xf(x) = x\ln (a-x)\),所以 \(y' = \ln (a-x) - \frac{x}{a-x}\),因为 \(x = 0\) 是该函数的极值点,所以 \(y'|_{x=0}=0\).

\(x=0\) 代入得,\(\ln a = 0 \Rightarrow a = 1\),需要注意的是这里并不能结束,因为导函数的零点是 变号零点 才是该点是原函数极值点的充分必要条件,接下来证充分性.

\(a = 1\) 时,\(y' = \ln (1-x) - \frac{x}{1-x}, \; y'' = \frac{-1}{1-x} - \frac{1}{(1-x)^2} = \frac{x-2}{(x-1)^2}\).

\(x \in (-\infty, 1)\) 时,\(y'' < 0\),此时 \(y'\) 单调递减,又因为 \(y'|_{x=0}=0\),所以 \(y\)\((-\infty, 0)\) 上单调递增,在 \((0, 1)\) 上单调递减,所以 \(x=0\) 是其极值点,所以 \(a=1\).

\((2).\) 先证 \(g(x) > 0\),考虑 \(g(x) = \frac 1x + \frac {1}{f(x)} = \frac 1x + \frac{1}{\ln(1-x)}\).

构造函数 \(h(x) = x + \ln(1-x)\),则 \(h'(x) = 1 - \frac {1}{1-x} = \frac {x}{x-1}\),所以当 \(x = 0\) 时,\(h'(x) = 0\)\(h(x)\) 取得极值.

所以当 \(x \in (-\infty, 0)\) 时,\(h'(x) > 0\)\(h(x)\) 单调递增,当 \(x \in (0, 1)\) 时,\(h'(x) < 0\)\(h(x)\) 单调递减,所以 \(h(x) \leq 0 \Leftrightarrow \ln(1-x) \leq -x\).

所以 \(g(x) = \frac 1x + \frac{1}{\ln (1-x)} > \frac 1x + \frac {1}{-x} = 0\),因为 \(x \neq 0\),所以无法取得等号. 再证 \(g(x) < 1\).

\((1)\) 知,\(y = xf(x)\)\((-\infty, 0)\) 上单调递增,在 \((0, 1)\) 上单调递减,所以 \(y \leq y|_{x=0} = 0\),所以要证 \(g(x) < 1\),即证 \(x+f(x)>xf(x)\).

\(1 - x = t \Rightarrow x = 1-t\),因为 \(x \in (-\infty, 0) \cup (0, 1)\),所以 \(t \in (0, 1) \cup (1, +\infty)\).

那么构造函数 \(\varphi(t) = t\ln t - t + 1\),所以 \(\varphi'(t) = \ln t\),所以当 \(t = 1\) 时,\(\varphi'(t) = 0\)\(\varphi(t)\) 取得极值.

\(t \in (0, 1)\) 时,\(\varphi'(t) < 0\)\(\varphi(t)\) 单调递增,当 \(t \in (1, +\infty)\) 时,\(\varphi'(t) < 0\)\(\varphi(t)\) 单调递减.

所以 \(\varphi(t) > \varphi(1) = 0\),所以 \(t\ln t - t + 1 > 0 \Leftrightarrow x + (1-x)\ln (1-x) > 0\),即 \(x+f(x) > xf(x)\),原不等式得证. 综上 \(0 < g(x) < 1\).

2021/09/04: 广东省2022届高三综合能力测试(一)

圆锥曲线

\(T_{21}.\) 已知双曲线 \(C: \frac{x^2}{a^2} - \frac{y^2}{b^2} = 1(a,b > 0)\) 的右焦点 \(F(2, 0)\),一条渐进线方程为 \(x - \sqrt 3y=0\).

\((1).\)\(C\) 的方程.
\((2).\)\(C\) 的左右顶点分别为 \(A,B\),过 \(F\) 的直线 \(l\) 交双曲线的右支于 \(M,N\) 两点,连接 \(M,B\) 交直线 \(x = \frac 32\) 于点 \(Q\),求证: \(A,Q,N\) 三点共线.


思考了半天是不是要找高考前的模拟题,突然觉得还是做做新题比较好(以后的也尽量找 2022 份的题,而且最近因为 [数据删除] 比较忙,所以就那这套卷子氵一下(

\((1).\) 由题意知,\(\frac ba = \frac{\sqrt 3}{3}, c = 2\),又因为 \(a^2 + b^2 = c^2\),所以 \(a = \sqrt 3, b = 1\),所以 \(C: \frac{x^2}{3} + y^2 = 1\).

\((2).\)\(MN: x = ty+2, M(x_1,y_1), N(x_2,y_2)\),联立 \(MN\) 和双曲线方程,得到 \((t^2 - 3)y^2 + 4ty + 1 = 0, \Delta = 12(t^2 + 1) > 0\).

通过图象观察得当 \(MN\) 与右支有两个交点时这两点纵坐标之积必为负数(证明可以通过 \(MN\)\(F\)),所以 \(y_1y_2 = \frac{1}{t^2-3} < 0 \Rightarrow t \in (-\sqrt 3, \sqrt 3)\).

因为 \(B(\sqrt 3, 0)\),所以 \(k_{MB} = \frac{y_1}{x_1 - \sqrt 3}\),所以 \(MB: y = \frac{y_1}{x_1 - \sqrt 3}(x - \sqrt 3)\),令 \(x = \frac 32\),得到 \(Q \left(\frac 32, \frac{(\frac 32 - \sqrt 3)y_1}{x_1-\sqrt 3} \right)\).

又因为 \(A(-\sqrt 3, 0)\),所以 \(\overrightarrow{AQ} = \left( \frac 32 + \sqrt 3, \frac{(\frac 32 - \sqrt 3)y_1}{x_1-\sqrt 3} \right), \overrightarrow{AN} = (x_2 + \sqrt 3, y_2)\).

要证 \(A,Q,N\) 三点共线,即证 \(\overrightarrow{AQ} = \lambda \overrightarrow{AN} \, (\lambda \in \mathbb {R})\),即证:

\[\begin{aligned} & \left( \frac 32 + \sqrt 3\right)y_2 = (x_2 + \sqrt 3)\left( \frac{(\frac 32 - \sqrt 3)y_1}{x_1-\sqrt 3}\right) \\ & \Leftrightarrow \frac{(1.5 + \sqrt 3) y_2 (ty_1 + 2 - \sqrt 3) - (1.5 - \sqrt 3) y_1 (ty_2 + 2 + \sqrt 3)}{ty_1 + 2 - \sqrt 3} = 0 \\ & \Leftrightarrow \frac{2\sqrt 3 y_1y_2 + \frac {\sqrt 3}{2} (y_1 + y_2)}{ty_1 + 2 - \sqrt 3} = 0 \Leftrightarrow 0 = 0 \end{aligned}\]

所以,\(\overrightarrow{AQ} = \lambda \overrightarrow{AN} \, (\lambda \in \mathbb {R})\)\(A,Q,N\) 三点共线.

函数导数

\(T_{22}.\) 已知函数 \(f(x) = \frac 12(x-1)^2+a(x - \ln x), g(x) = \frac 13(x-1)^3 - f(x)\).

\((1).\) 确定 \(a\) 的所有值,使函数 \(f(x)\)\((0, +\infty)\) 上单调递增.
\((2).\) 若函数 \(g(x)\)\(x = x_1\)\(x = x_2\) 处取得极小值 \(g(x_1)\)\(g(x_2)\),证明: \(g(x_1)+g(x_2)+3a+\frac 12<0\).


其实这个题质量还蛮不错的,第二问的证明过程需要比较长的代数化简,同时考察了灰色地带的立方和公式,但是总体思路还是比较简单的(

\((1).\) \(f(x)\) 的定义域为 \((0, +\infty)\),由题意知 \(f'(x) = x - 1 + a(1-\frac 1x) = (x-1) + \frac ax(x-1) = \frac{(x-1)(x+a)}{x}\).

\(a \ge 0\) 时,当 \(x = 1\) 时,\(f'(x) = 0\)\(f(x)\) 取得极值. 当 \(x \in (0,1)\) 时,\(f'(x) < 0\)\(f(x)\) 单调递减,不符合题意.

\(-1 < a < 0\) 时,当 \(x = 1\)\(x = a\) 时,\(f'(x) = 0\)\(f(x)\) 取得极值. 当 \(x \in (-a, 1)\) 时,\(f'(x) < 0\)\(f(x)\) 单调递减,不符合题意.

\(a < -1\) 时,当 \(x = 1\)\(x = a\) 时,\(f'(x) = 0\)\(f(x)\) 取得极值. 当 \(x \in (1, -a)\) 时,\(f'(x) < 0\)\(f(x)\) 单调递减,不符合题意.

\(a = -1\) 时,\(f'(x) = \frac{(x-1)^2}{x} \geq 0\),所以 \(f(x)\)\((0, +\infty)\) 上单调递增,符合题意. 综上,\(a = -1\).

\((2).\) 由题意知,\(g(x) = \frac 13(x-1)^3 - \frac 12(x-1)^2 + a(x - \ln x)\),所以 \(g'(x) = \frac{(x-1)(x^2-2x-a)}{x}\).

因为 \(g(x)\) 存在两个极小值点,所以方程 \(g'(x) = 0\) 有且仅有三个实数解,所以 \(\Delta = 4+4a > 0 \Rightarrow a > -1\).

所以当 \(x = 1\)\(x = -1 \pm \sqrt{a+1}\) 时,\(g'(x) = 0\)\(g(x)\) 取得极值,又因为 \(g(x)\) 定义域为 \((0, +\infty)\),所以 \(1 - \sqrt{a+1} > 0 \Rightarrow a \in (-1, 0)\).

所以当 \(x \in (0, 1 - \sqrt{a+1})\) 时,\(g'(x) < 0\)\(g(x)\) 单调递减,当 \(x \in (1 - \sqrt{a+1}, 1)\) 时,\(g'(x) > 0\)\(g(x)\) 单调递增.

\(x \in (1, 1 + \sqrt{a+1})\) 时,\(g'(x) < 0\)\(g(x)\) 单调递减,当 \(x \in (1 + \sqrt{a+1}, +\infty)\) 时,\(g'(x) > 0\)\(g(x)\) 单调递增.

所以 \(1 - \sqrt{a+1}, 1 + \sqrt{a+1}\)\(g(x)\) 的极小值点,不妨设 \(x_1<x_2\),又因为 \(x_1,x_2\) 是方程 \(x^2-2x-a=0\) 的两个根,所以 \(x_1+x_2=2, x_1x_2=-a\).

\[\begin{aligned} g(x_1) + g(x_2) + 3a + \frac 12 & = \frac 13 \left[ (x_1 - 1)^3 + (x_2 - 1)^3\right] - f(x_1) - f(x_2) + 3a + \frac 12 \\ & = \frac 13 (x_1+x_2-2)\left[ (x_1)^2 + (x_1-1)(x_2-1) + (x_2)^2\right] - \frac 12 \left[ x_1^2 + x_2^2 -2(x_1+x_2)+2\right] - a[(x_1+x_2) - \ln x_1x_2] + 3a + \frac 12 \\ & = -a-1-a[2 - \ln (-a)] + 3a + \frac 12 = a\ln (-a) - \frac 12 \end{aligned} \]

构造函数 \(h(x) = -x\ln x - \frac 12\),所以 \(h'(x) = -1-\ln x\),所以当 \(x = \frac 1e\) 时,\(h'(x) = 0\)\(h(x)\) 取得极值.

\(x \in \left(0, \frac 1e \right)\) 时,\(h'(x) > 0\)\(h(x)\) 单调递增,当 \(x \in \left( \frac 1e, +\infty\right)\) 时,\(h'(x) < 0\)\(h(x)\) 单调递减.

所以 \(h(x) \leq h\left( \frac 1e\right) = \frac 1e - \frac 12 < 0\),令 \(x = -a\),所以 \(a\ln (-a) - \frac 12 < 0\),所以原命题得证.

2021/09/05: 雅礼中学 2022 届高三入学考试

数列 & 圆锥曲线

\(T_{8}\)\(f(x)\) 是定义在 \(\mathbb {R}\) 上的奇函数,满足 \(f(2-x) = f(x)\),数列 \(\{a_n\}\) 满足 \(a_1 = -1\),且 \(a_{n+1} = \left( 1 + \frac 1n\right)a_n + \frac 2n (n \in \mathbb{N^*})\),则 \(f(a_{22})=\underline{\quad\quad\quad}\).


类似于同构化的操作,感觉是显然的,然后就是个裂项相消,但是因为 \(f(x)\) 的条件给的太简单了,所以考场上很多老哥直接盲选估计都行(((

由题意知 \(a_{n+1} = \left(\frac{n+1}{n}\right)a_n + \frac 2n\),所以 \(\frac{a_{n+1}}{n+1} = \frac{a_n}{n} + \frac{2}{n(n+1)}\).

\(b_n = \frac{a_n}{n}\),所以 \(b_{n+1} = b_n + \frac{2}{n(n+1)} = b_1 + 2\left[ \frac{1}{1 \times 2} + \frac{1}{2 \times 3} + \cdots + \frac{1}{n(n+1)}\right] = \frac{n-1}{n+1}\),所以 \(a_n = nb_n = n - 2\),所以 \(a_{22} = 20\).

又因为 \(f(2-x) = f(x)\)\(f(x) + f(-x) = 0\),所以反复迭代得到 \(f(20) = f(18) = \cdots = f(0) = 0\).


\(T_{16}.\) 某校数学课外小组在坐标纸上为学校的一块空地设计植树方案,方案如下: 第 \(k\) 棵树的坐标为 \(P_k(x_k,y_k)\),其中 \(x_1 = 1, y_1 = 1\),当 \(x \geq 2\) 时,满足\(\begin{cases} x_k = x_{k-1} + 1 - 5\left( \left[\frac{k-1}{5} \right] - \left[ \frac{k-2}{5}\right] \right) \\ y_k = y_{k-1}+\left[\frac{k-1}{5} \right] - \left[ \frac{k-2}{5}\right] \end{cases}\),其中 \([x]\)\(\rm Gauss\) 取整函数,\({\rm e.g.}\;[\pi] = 3, [\ln 2] = 0\),按此方案则第 \(2008\) 棵树的坐标为 \(\underline{\quad\quad\quad}\).


虽然看起来是个数列题,但是感觉像是个数论题(?,不过感觉搞得还是太 \(\rm ez\) 了.

注意到 \(\left[\frac{k-1}{5} \right] - \left[ \frac{k-2}{5}\right]\) 这个式子只有在 \(k \equiv 1 \pmod{5}\) 的时候才为 \(1\),其他时候为 \(0\).

所以 \(x_k = \begin{cases} x_{k-1} - 4 & k \equiv 1 \pmod{5} \\ x_{k-1} + 1 & {\rm otherwise}\end{cases}, y_k = \begin{cases} y_{k-1} & k \equiv 1 \pmod{5} \\ y_{k-1} + 1 & {\rm otherwise}\end{cases}\).

写出这两个数列的前几项就可以发现规律: \(\{ x_k\}\) 是一个以 \(1,2,3,4,5\) 为循环节的数列,而 \(\{ y_{5k+1}\}\) 是一个以 \(2\) 为首项 \(1\) 为公差的等差数列.

又因为 \(\left[ \frac{2008}{5}\right] = 401\)\(2008 \equiv 3 \pmod{5}\),所以第 \(2008\) 棵树的坐标为 \((3, 402)\).


\(T_{21}.\) 已知椭圆 \(C: \frac{y^2}{a^2} + \frac{x^2}{b^2} = 1(a > b > 0)\) 的焦距与椭圆 \(\frac{x^2}{3} + y^2 = 1\) 的焦距相等,且 \(C\) 经过抛物线 \(y = (x-1)^2 + \sqrt 2\) 的顶点.

\((1).\) 求椭圆 \(C\) 的方程.
\((2).\) 若直线 \(y = kx+m\)\(C\) 相交于 \(A,B\) 两点,且 \(A,B\) 关于直线 \(x+ty+1=0\) 对称,\(O\)\(C\) 的对称中心,且 \(\triangle ABC\) 的面积为 \(\frac{\sqrt{10}}{3}\),求 \(k\) 的值.


好家伙,这个第二问给我算吐了,第一遍写错 \(x_1+x_2\) 最后代数式复杂的离谱,第二遍三角形面积忘记 \(\frac 12\),感觉自己已经垃圾得可以入土了/kk

\((1).\) 由题意知,因为两椭圆焦距相同,所以 \(c = \sqrt{(\sqrt 3)^2 - 1^2} = \sqrt 2\),又因为 \(C\) 过点 \((1, \sqrt 2)\),所以 \(2b^2 + a^2 = a^2b^2\),又 \(a^2+b^2=c^2\),解得 \(a = 2, b = \sqrt 2\),所以 \(C: \frac{y^2}{4}+\frac{x^2}{2}=1\).

\((2).\) 联立直线 \(AB\) 和椭圆 \(C\) 的方程,得到 \((k^2+2)x^2+2mkx+m^2-4=0\),所以 \(x_1+x_2 = \frac{-2mk}{k^2+2}, x_1x_2 = \frac{m^2-4}{k^2+2}\).

因为 \(A,B\) 关于直线 \(x+ty+1=0\) 对称,所以 \(AB\) 与之垂直且该直线过线段 \(AB\) 中点 \(P\),所以 \(k \cdot -\frac 1t = -1 \Rightarrow k=t\) \(P\left(\frac{x_1+x_2}{2}, \frac{y_1+y_2}{2} \right)\)\(P\left( \frac{-mk}{k^2+2}, \frac{2m}{k^2+2}\right)\).

代入直线得 \(\frac{-mk}{k^2+2} + \frac{2mk}{k^2+2}+1=0 \Leftrightarrow k^2+mk+2=0 \Rightarrow m = -\frac{k^2+2}{k}\). 所以联立方程 \(\Delta = 8\left( k^2 - \frac{4}{k^2}\right) > 0 \Rightarrow -\sqrt 2 < k < \sqrt 2\).

所以 \(|AB| = \sqrt{1+k^2} |x_A - x_B| = \sqrt{1+k^2} \frac{\sqrt{\Delta}}{|a|} = \sqrt{1+k^2} \frac{2\sqrt{2(k^2+1)(k^2-2)}}{k\sqrt{k^2+2}}\)\(d_{O\to AB} = \frac{|m|}{\sqrt{k^2+1}}\).

所以 \(S_{\triangle OAB} = \frac 12 d|AB| = \frac{\sqrt{2(k^2-4)}}{k^2} = \frac{\sqrt{10}}{3} \Rightarrow k = \pm \sqrt 3\).

函数导数

\(T_{22}.\) 已知函数 \(f(x) = x\ln x - \frac 12x^3 + a, g(x) = xe^{1-x} + \frac{2a-1}{2}x^3-x(a \in \mathbb{R})\).

\((1).\) 若函数 \(f(x)\)\(\left( \frac 1e, 1\right)\) 上有零点,求 \(a\) 的取值范围.
\((2).\)\(x \geq 1\) 时,不等式 \(f(x) \leq g(x)\) 恒成立,求实数 \(a\) 的范围.


其实必要性探路这件事吧,还是有点玄乎的,但是看到点复杂的指对幂混合呢,一不小心用一用还是蛮方便的((

\((1).\) 由题意知 \(f'(x) = \ln x + 1 - \frac 32 x^2\)\(f''(x) = \frac 1x - 3x = \frac{1-3x^2}{x}\),所以当 \(x = \frac {\sqrt 3}{3}\) 时,\(f''(x) = 0\)\(f'(x)\) 取得极值.

所以当 \(x \in \left(0, \frac{\sqrt 3}{3} \right)\) 时,\(f''(x) > 0\)\(f'(x)\) 单调递增,当 \(x \in \left(\frac{\sqrt 3}{3}, +\infty \right)\) 时,\(f''(x) < 0\)\(f'(x)\) 单调递减.

所以 \(f'(x) \leq f' \left(\frac{\sqrt 3}{3} \right) = \ln \frac{1}{\sqrt 3} - \frac 12 = \ln \frac{1}{\sqrt 3} - \ln \sqrt e < 0\),所以在 \((0, +\infty)\) 上,\(f'(x)\) 恒负,\(f(x)\) 单调递减.

所以当 \(f\left( \frac 1e\right)f(1) < 0\) 时,\(f(x)\)\(\left( \frac 1e, 1\right)\) 上有零点,解得 \(a \in \left( \frac {1}{2e^3} + \frac 1e, \frac 12\right)\).

\((2).\) 由题意知,\(f(x) \leq g(x) \Leftrightarrow x\ln x + a \leq xe^{1-x} + ax^3 - x \Leftrightarrow e^{1-x}+ax^2-\ln x -1-\frac{a}{x} \geq 0\).

构造函数 \(h(x) = e^{1-x}+ax^2-\ln x - 1 -\frac{a}{x}\),则 \(h'(x)=-e^{1-x}+2ax-\frac 1x+\frac{a}{x^2}\).

因为当 \(x \geq 1\) 时,\(h(x) \geq 0\) 恒成立,所以 \(h'(1) \geq 0\),即 \(-1+2a-1+a \geq 0 \Rightarrow a \geq \frac 23\).

接着证充分性,当 \(a \geq \frac 23\) 时,此时 \(h''(x) = e^{1-x}+\frac {1}{x^2}+2a(1 - \frac {1}{x^3}) > 0\) 恒成立,那么 \(h'(x)\)\([1, +\infty)\) 上单调递增.

所以 \(h'(x) \geq h'(1) = 3a - 2 > 0\),所以 \(h(x)\)\([1,+\infty)\) 上单调递增,所以 \(h(x) \geq h(1) = 0\),所以 \(f(x) \leq g(x)\) 得证. 综上 \(a \geq \frac 23\).

TO BE CONTINUED

posted on 2021-08-30 17:42  Herself32  阅读(75)  评论(0编辑  收藏  举报

导航